"Lagrangian formulation of electromagetism"의 두 판 사이의 차이

수학노트
둘러보기로 가기 검색하러 가기
imported>Pythagoras0
imported>Pythagoras0
2번째 줄: 2번째 줄:
 
* 전기장 $\mathbf{E}=\nabla \phi$
 
* 전기장 $\mathbf{E}=\nabla \phi$
 
* 자기장 $\mathbf{B}=\nabla \times \mathbf{A}$
 
* 자기장 $\mathbf{B}=\nabla \times \mathbf{A}$
* 전자기장 안에 놓인 하전입자에 대한 라그랑지안
+
* 전자기장 안에 놓인 질량 $m$, 전하 $e$의 하전입자에 대한 라그랑지안
 
:<math>L(q,\dot{q})=\frac{m||\dot{q}||^2}{2}-e\phi+eA_{i}\dot{q}^{i}</math>
 
:<math>L(q,\dot{q})=\frac{m||\dot{q}||^2}{2}-e\phi+eA_{i}\dot{q}^{i}</math>
 
* 켤레운동량
 
* 켤레운동량
 
:<math>p_{i}=\frac{\partial{L}}{\partial{\dot{q}^{i}}}=m \dot{q}_{i}+eA_{i}=mv_{i}+eA_{i}</math>
 
:<math>p_{i}=\frac{\partial{L}}{\partial{\dot{q}^{i}}}=m \dot{q}_{i}+eA_{i}=mv_{i}+eA_{i}</math>
 +
* 오일러-라그랑지 방정식 <math>\dot{p}=F</math>은 다음과 같이 쓰여진다
 
$$
 
$$
\dot{p}_{i}=m\frac{dv_{i}}{dt}+e\frac{\partial{A_{i}}}{\partial t}+e\frac{\partial{A_{i}}}{\partial{q}^{j}}\dot{q}^{j}=m\frac{dv_{i}}{dt}+e\frac{\partial{A_{i}}}{\partial{q}^{j}}\dot{q}^{j}
+
\dot{p}_{i}=m\frac{dv_{i}}{dt}+e\frac{\partial{A_{i}}}{\partial t}+e\frac{\partial{A_{i}}}{\partial{q}^{j}}\dot{q}^{j}=m\frac{dv_{i}}{dt}+e\frac{\partial{A_{i}}}{\partial{q}^{j}}\dot{q}^{j} \\
$$
 
$$
 
 
F_{i}=\frac{\partial{L}}{\partial{q^{i}}}=\frac{\partial}{\partial{{q}^{i}}}(-e\phi+eA_{j}\dot{q}^{j})=-e\frac{\partial{\phi}}{\partial{q}^{i}} +e\frac{\partial{A_{j}}}{\partial{q}^{i}}\dot{q}^{j}
 
F_{i}=\frac{\partial{L}}{\partial{q^{i}}}=\frac{\partial}{\partial{{q}^{i}}}(-e\phi+eA_{j}\dot{q}^{j})=-e\frac{\partial{\phi}}{\partial{q}^{i}} +e\frac{\partial{A_{j}}}{\partial{q}^{i}}\dot{q}^{j}
 
$$
 
$$
* 오일러-라그랑지 방정식 <math>\dot{p}=F,</math>
+
:<math>m\frac{dv_{i}}{dt}=eE_{i}+eF_{ij}\dot{q}^{j},\quad i=1,2,3 \label{eom}</math>
:<math>m\frac{dv_{i}}{dt}=eE_{i}+eF_{ij}\dot{q}^{j}</math>
+
여기서 $F_{\mu\nu} = \partial_\mu A_\nu - \partial_\nu A_\mu \,\!$
where $F_{\mu\nu} = \partial_\mu A_\nu - \partial_\nu A_\mu \,\!$
+
* $F_{12}=B_{3}$, $F_{23}=B_{1}$, $F_{31}=B_{2}$
* for example, if $i=1$
+
* 가령 $i=1$이면, \ref{eom}은 다음과 같다
 
$$
 
$$
 
ma_1=eE_1+e(F_{11}\dot{q}^{1}+F_{12}\dot{q}^{2}+F_{13}\dot{q}^{3})=eE_1+e(F_{12}\dot{q}^{2}-F_{31}\dot{q}^{3})=eE_1+e(\mathbf{v}\times \mathbf{B})_{1}
 
ma_1=eE_1+e(F_{11}\dot{q}^{1}+F_{12}\dot{q}^{2}+F_{13}\dot{q}^{3})=eE_1+e(F_{12}\dot{q}^{2}-F_{31}\dot{q}^{3})=eE_1+e(\mathbf{v}\times \mathbf{B})_{1}
 
$$
 
$$
where $F_{12}=B_{3}$ and $F_{31}=B_{2}$
+
* 전하가 받는 힘 $\mathbf{F}$는 다음과 같다
* 이를 로렌츠 힘이라 한다
+
:<math>\mathbf{F}=e\mathbf{E}+e\mathbf{v}\times \mathbf{B}</math>
* 전하가 받는 힘은
+
* 이 중에서 $e\mathbf{v}\times \mathbf{B}$를 로렌츠 힘이라 한다
:<math>e\mathbf{E}+e\mathbf{v}\times \mathbf{B}</math>
 
 
* http://en.wikipedia.org/wiki/Lorentz_force
 
* http://en.wikipedia.org/wiki/Lorentz_force
 
* {{수학노트|url=전자기_텐서와_맥스웰_방정식}}
 
* {{수학노트|url=전자기_텐서와_맥스웰_방정식}}

2014년 1월 29일 (수) 06:35 판

하전입자에 대한 라그랑지안

  • 전기장 $\mathbf{E}=\nabla \phi$
  • 자기장 $\mathbf{B}=\nabla \times \mathbf{A}$
  • 전자기장 안에 놓인 질량 $m$, 전하 $e$의 하전입자에 대한 라그랑지안

\[L(q,\dot{q})=\frac{m||\dot{q}||^2}{2}-e\phi+eA_{i}\dot{q}^{i}\]

  • 켤레운동량

\[p_{i}=\frac{\partial{L}}{\partial{\dot{q}^{i}}}=m \dot{q}_{i}+eA_{i}=mv_{i}+eA_{i}\]

  • 오일러-라그랑지 방정식 \(\dot{p}=F\)은 다음과 같이 쓰여진다

$$ \dot{p}_{i}=m\frac{dv_{i}}{dt}+e\frac{\partial{A_{i}}}{\partial t}+e\frac{\partial{A_{i}}}{\partial{q}^{j}}\dot{q}^{j}=m\frac{dv_{i}}{dt}+e\frac{\partial{A_{i}}}{\partial{q}^{j}}\dot{q}^{j} \\ F_{i}=\frac{\partial{L}}{\partial{q^{i}}}=\frac{\partial}{\partial{{q}^{i}}}(-e\phi+eA_{j}\dot{q}^{j})=-e\frac{\partial{\phi}}{\partial{q}^{i}} +e\frac{\partial{A_{j}}}{\partial{q}^{i}}\dot{q}^{j} $$ \[m\frac{dv_{i}}{dt}=eE_{i}+eF_{ij}\dot{q}^{j},\quad i=1,2,3 \label{eom}\] 여기서 $F_{\mu\nu} = \partial_\mu A_\nu - \partial_\nu A_\mu \,\!$

  • $F_{12}=B_{3}$, $F_{23}=B_{1}$, $F_{31}=B_{2}$
  • 가령 $i=1$이면, \ref{eom}은 다음과 같다

$$ ma_1=eE_1+e(F_{11}\dot{q}^{1}+F_{12}\dot{q}^{2}+F_{13}\dot{q}^{3})=eE_1+e(F_{12}\dot{q}^{2}-F_{31}\dot{q}^{3})=eE_1+e(\mathbf{v}\times \mathbf{B})_{1} $$

  • 전하가 받는 힘 $\mathbf{F}$는 다음과 같다

\[\mathbf{F}=e\mathbf{E}+e\mathbf{v}\times \mathbf{B}\]


전자기장에 대한 라그랑지안

free

  • 상호작용이 없는 전자기장의 라그랑지안은 다음과 같다

$$\mathcal{L}_{\text{EM}}= - \frac{1}{4}F_{\mu\nu}F^{\mu\nu}=\frac{1}{2}(\mathbf{E}^2-\mathbf{B}^2)$$ 이 때 \(F_{\mu\nu} = \partial_\mu A_\nu - \partial_\nu A_\mu \,\!\)는 전자기텐서, $A=(A_{\mu})$는 전자기 포텐셜

  • action

\[S=-\frac{1}{4}\int F^{\alpha\beta}F_{\alpha\beta}\,d^{4}x\]

  • 라그랑지안은 전자기 포텐셜의 다음과 같은 변환에 대하여 불변이다

\[A_{\mu}(x) \to A_{\mu}(x)-\partial_{\mu}\Lambda(x)\] 여기서 $\Lambda(x)$는 임의의 스칼라장

  • equation of motion

$$ \partial_\mu F^{\mu\nu}=0 $$

in the presence of $j$ and $\rho$

  • Lagrangian

$$L=-\frac{1}{4}F_{\mu\nu}F^{\mu\nu}-ej_\mu A^\mu$$

  • action

$$S[\phi,A]=\int_{t_1}^{t_2}\int_{\mathbb{R}^3}\left(-\rho\phi+j\cdot A+\frac{\epsilon_0}{2}E^2-\frac{1}{2\mu_0}B^2\right)\,dV\,dt$$

  • w.r.t $\phi$

$$\nabla\cdot E=\frac{\rho}{\epsilon_0}$$

  • w.r.t $A$

$$\nabla\times B=\mu_0j+\epsilon_0\mu_0\frac{\partial E}{\partial t}$$



memo


related items


매스매티카 파일 및 계산 리소스


리뷰, 에세이, 강의노트


questions